You are on page 1of 24

Kirchhoff's Laws Contents: 2.1 2.2 2.3 2.4 2.5 2.6 2.7 2.8 2.9 2.9.1 2.9.2 2.

10 2.11 2.12 2.13 2.14 2.15 Kirchhoffs Current Law (KCL) Nodes and Branches Single Node-Pair Circuit Equivalent of Parallel resistors. Current Division Kirchhoffs Voltage Law Equivalent Resistance of a Series Path Voltage Division Use of series Parallel combinations Standard Values Circuit Solution Bridge Circuits Delta-Wye Transformations Dependent sources More on Current and voltage dividers Use of Kirchhoffs Laws in Analysis Inter node Currents Problems 1 3 4 4 6 7 9 10 11 11 12 13 14 16 17 18 19 21

2.0 Together with Ohms Law, Kirchhoffs Laws (entirely commonsense laws again) comprise the backbone of Electrical Engineering. These are two in number. The first one, called Kirchhoffs Current Law (KCL), deals with flow of charges. The second one deals with voltages and is called Kirchhoffs Voltage Law (KVL). Kirchhoffs Current Law (KCL)

2.1 In laymans words it says: Whatever current flows into a point (or part of a circuit) must leave it also. It can be reworded to say: net current into a point (or part of a circuit) is zero, where net means sum with due regard to signs. Mathematically, Algebraic sum of currents into a point (or part of a circuit) is zero. Iin=0 (Algebraic) (2.1)

It is also true with into replaced with out of. Iout=0 (Algebraic) (2.2)

For example in fig 2-1, we can use eq (2.2) to write: Iin =4 -3 + 8 -9=0, Where we have added currents entering as positive and currents flowing out as negative. We could also write Iout =-4+3-8+9 =0 Another way of saying the same is Sum of currents flowing in = Sum of currents flowing out) Important: KCL is applicable to a point, part of a circuit, area, or a volume in space, eg your classroom. Example 2-1 Verify KCL on fig 2-2 Solution Iin=0 ? Iin= 4-3+8-10= -1 0 This diagram fails the KCL test; hence it is an incorrect diagram if it relates to a real circuit. Example 2-1 In fig 2-3 find Ix Solution Applying KCL: Iin=0 (All currents inwards will be taken as positive and those outwards as negative) 2-(-3)-5-Ix=0 Ix=2+3-5=0 Ix=0 Ans

Note : The same result will be obtained if we used either Iout=0, or Iin = Iout. Readers are encouraged to verify this. 2.2 Nodes and Branches

Node: A junction of two or more circuit elements is a current node, normally called node In figure 2-4, a, b, c, d, are all nodes. Note the extent of node d. Bottom ends the 4 and 5 resistors and of the 12V source along with the conductor joining them are all part of node d , being electrically the same point. Principal Node: A node where three or more circuit elements interconnect is called a Principal Node. Current partition takes place at a Principal Node. All other nodes are simple nodes. Nodes b and d are principal nodes. Path: Path has the same meaning as in English language, except that it is electrical. In fig 2-4, a-b is a path. So are a-b-c, a-b-d, b-c-d-b, a-b-c-d-a. Closed Path: b-c-d-b, a-b-c-d-a are closed paths in that they each end where they start. Paths a-b-c-d-a and b-c-d-ab are the same closed path because the two travel over the same nodes and same elements. Thus in a closed path it does not matter where you start as long as you end where you start. Can you find another closed path in this circuit? Branch: A path between two nodes and not crossing a principal node is a branch. Essentially what it implies is that whatever current enters a branch at one end the same current proceeds on to the other end. Paths a-b, a-d, b-c, b-d, cd, and b-c-d, b-a-d are all branches. However path a-b-c is not a branch because node b is a principal node. Similarly path a-b-d is not a branch because it crosses a principal node (b, again). Parallel Branches Between a pair of nodes there may be more than one branch eg b-a-d, b-d (though 5), and b-c-d, all lie between nodes b-d. All these branches are electrically parallel. Any current leaving node, say, b, will reach the other node unchanged. Parallel branches are parallel end to end. Branch d-b is not parallel (electrically) to branch d-c, although they look physically on paper. Single Node-Pair Circuit 2.3 In figure 2-5, the top ends of the voltage source, resistor R1, and resistor R2 are all connected to node a. Similarly, their bottom ends are connected to node b. The source causes a voltage across node-pair a-b of Vab=V volts. The same voltage is available across each of the resistors because those are connected between the nodes a,b. If there were any other branches across nodes a, b, those would also receive the same voltage. From the definitions above recall that all elements or branches connected across common nodes are in parallel. Each of these parallel branches draws current from the source according to Ohms Law. 2.4 Equivalent of Parallel resistors.

2.4.1 What is the resistance seen by the source as a combined effect of the two resistors in parallel? To answer this we find the current drawn from the source in fig 2-5. I=I1+I2 =V/R1 +V/R2 =V(1/ R1 +1/R2) Now we draw another circuit in figure 2-6 which we suppose is electrically equivalent to that of figure 2-5. Since V is driving both circuits, if the two circuits are equivalent, I in figure 2-6 must be equal to I in figure 2-5. In figure 2-6, I= V/R where R must be the equivalent of R1, R2 in parallel. V/R= I = V(1/ R1 +1/R2) 1/R= (1/ R1 +1/R2) By induction, this law is extended to N resistors in parallel. 1/R=N(1/Rx) (2-3) (KCL)

The reciprocal of equivalent resistor of a parallel combination is the sum of reciprocals of individual resistors in the combination. Example 2-2 Three resistors of values 10 , 30 and 60 are connected in parallel. Find the equivalent resistor. Solution Here n=3, x=1,2,3 If R is the equivalent resistor of the parallel combination: 1/R= 1/10+1/30+1/60 =3/20 R=20/3 =6.67 Tips: Beginners often forget the reciprocation, and write R= 3/20 which is incorrect. Beware! The total current I of a parallel combination is sum of all individual currents, hence more than current of any one branch. As a result, the equivalent resistance of the combination is less than any of the branch resistanceseven the lowest of them. Verify this in the above example. Conductances in Parallel 2.4.2 Recall G= 1/R where G is in Siemens and R is in Ohms

Equivalent combination of parallel resistors can be written as:

1/R=n(1/Rx) G= n(Gx) PP 2-1

(from equation 2-3) (2-4)

Two resistors of 3 and 2 are in parallel with a conductor of 1/6 Siemen. What is the equivalent resistance and conductance of the combination? (Ans: 1 , 1S) Ideal Sources in Parallel 2.4.3 Parallel means same voltage across all parallel elements. Hence ideal voltage sources cannot be connected in parallel however small the voltage difference between them may be. Current sources can be connected in parallel and the net current out will be the algebraic sum of the individual currents.

Current Division 2.5 A parallel combination of N conductors (or resistors) makes a current divider on the total input current I. Fig 2-7). V is the common voltage across the branches. By Ohms Law, I=GV If Gx is the conductance of the xth branch, current in xth branch is given by: Ix = Gx V, (1 x I= NIx N)

=
Ix=

=
(2-5)

At this stage dont you think the Current Division Rule, (CDR) is simply a question of proportionate shares out of the total available? Or does it remind you of the division of meat of a sacrificial animal (cow, buffalo, camel etc) being divided among the share holders in proportion to their respective paid shares? Each parallel branch gets a current proportional to its share in the total conductance. In normal practice we mostly deal with resistors rather conductors. Fig 2-8(a) Simply put G= Then: Ix =I

than

in equation 2-5

(2-6)

Equations 2-5 and 2-6 are alternate forms of the Current Division Rule, (CDR), and a circuit consisting of parallel branches is called a current divider.

Example 2-3 (a) In figure 2-7(b), R1=4, R2=16, R3=64 . Total Current I= 2.1 A. Find the current in each resistor. (b) In figure 2-8 (b) find the value of I4/ I. Solution (a): G=1/R=1/4+1/16+1/64 =(16+4+1)/64 G3=1/R3 Using (2-6) =21/64 Siemen

=
I1= 2.1 x ( )

=
=1.6 A

I2= 2.1 (G2/G) =2.1 x [(1/16) / (21/64)]=2.1 x (4/21)= 0.4 A Similarly the share of G3 in the total current is : 2.1 x (G3/G) =2.1 [(1/64)/(21/64)]=0.1 A A comment on the results of example 2-3 is useful: The largest resistor (=smallest conductance), has the least share of current, and vice versa. The intermediate value conductance (G2 in this case) has an intermediate share of current. Solution (b): Current I, whose value is determined by the overall circuit, flows into node A (fig 2-8(a), where it branches into I1 and I2. I1 further subdivides into I3 and I4 at node B.

.
(2-5)

By CDR: Ix=

The ratio of I1 to I is not that straight forward to calculate. I2 flows in the 6 resistor, but I1 flows through a combination of [3 s (18 p 9)] = 9. None of the four resistances involved can alone determine the proportion. Rather, the effective conductances (or resistances) of the two branches carrying I1, I2 together determine the proportion.

=
Consequently,

= .

This example emphasizes the rule that current proportioning in a voltage divider depends on the respective path resistances, rather than just the resistors touching the node. The current divider exists between two nodes encompassing the parallel paths of the divider. There are three resistive dividers. One formed by 18 and 9 in parallel between nodes B, C; the other by the two paths between nodes A and C; and the third by the resistances 12 and 24 in parallel.

Current I1 flows through 3 resistor and at node B (Fig 2-22y) divides into I2 through 9 and I3 through 18 resistor. At node C the currents recombine to give back I1. Thus the division is nullified here. The divider lies between nodes B and C, and the division is controlled by the branches (single elements in this case) connected between the two nodes , viz, 9 and 18.

The 3 resistor, and indeed any other resistor does not affect the proportion of I2and I3 , although they may very well affect the valueof I1 itself.
At the junction of 5 with 3 and 6 resistors (node A in

Fig 2-22y) I1+I2=I. Current division is taking place here. This is one end of a current divider. Where does this divider end? What is the role of each of these three resistors in determining the current proportion? And is there any other element which affects the proportion of current division? To answer this question we must determine the boundaries of the current divider.. The 5 resistor will affect the value of I, being an element in the circuit of I, but it does not at all affect the proportion of current division. The reason is that a current division takes place between (two ormore) parallel branches. Obviously, the ends of the branches touch at two nodes. In this case these nodes are A and

C. Notice the current into node A is I and current out of I= I1+I2. I2=Vx/6 A

I1 Vx/3 Vx/9 Vx/18; Vx is applied across a complex combination of three resistors 3,9,18 having an equivalent resistance of 3+(9 p 18) =9. I1=Vx/9 A , and, I2= Vx/6 A

Kirchhoffs Voltage Law 2.6 Kirchhoffs Voltage Law (KVL for short) is another commonsense law. It applies to complete paths (Go round and come back to same point) in a circuit. Before we state KVL formally consider the following question. If we start climbing up (in a helicopter, say) and rise up by hu meters, and then descend by hd meters. If now we find that we are back to the original height, what is the obvious relation between h u, and hd? Obviously, hu-hd=0 ( net height change is zero) hd = hu In plain words, in order to revert to the original height, (or have a net zero change in height in a complete manoeuvre), we must descend as much as we rose. Similarly , in a circuit if we travel around a closed path---ie start from a given point and travelling over various elements or branches reach back at the same point--- our net potential gain (or voltage gain) must be zero. In passing over some elements, we will rise in potential, while over others we will fall in potential. The net rise (considering a fall as negative rise, and vice versa) must be zero, or equivalently, the net fall must be zero. We write: rise= 0 (over closed path, with due regard to signs), or fall= 0 (over closed path, with due regard to signs). (The word drop is often used for fall). To illustrate the point further, use figure 2-9(a). In travelling around the complete path we can start from anywhere. Let us say we start at the ve terminal of the 3v source. The direction can be either clockwise (CW) or counter clockwise (CCW). Suppose we choose to travel CW. From the ve terminal to the +ve terminal of the 3v source we have gone through a voltage rise of 3v volts provided by the source. It is a rise because we travel from a lower potential to a higher potential (from - to +). We continue and pass over the 3 resistor undergoing a voltage drop which we call VR, shown by the polarity (+,-) signs next to the resistor. The rise across the 3 is (VR) volt because VR is a drop along our path (we travelled from the+ sign to the - sign). The net rise until now is 3-VR volts. We travel further and over the block marked Y coming across another drop of VY which is a rise of (-VY) . The net rise until now is 3- VR Vx

Vx is another drop, but Vz is a rise because we are travelling from the (-) sign to the (+) sign. (The actual value and sign of VZ and VY is not known yet and does not matter). By this time we are back to the starting point and have closed (completed) the path. We write: 3-VR-VY-VX+VZ=0 We have closed the path despite the fact that there is an open switch en route through which no current can pass, and we know nothing about what is in block Y. KVL does not require a flow of current. It is applicable even in free space. Now we look at Fig 2.9(b). A single voltage source gives a rise of Vs volts from a to b. Vab = -Vs. It is an incomplete path. But we can still apply KVL involving Vs by closing the path in space. An infinite number of paths are available from b to a, and two such paths are abca, and abda. Around abca, rises: Vs-Vbca=0; Around abda, rises: Vs-Vbda=0; (Vbca is the drop from b to a via c) (Vzda is the drop from b to a via d)

Comparing the two we see that: Vbca= Vbda, where c, d were any randomly selected points in space. This means that the voltage drop from point b back to point a is the same whichever route is taken. It is to be noted again that: KVL is applicable even in free space.

The necessary condition in writing KVL is that you cater for all voltages on a route (any route and even a broken one) back to the starting point. More practice is needed here. Also problems with kvl on shared currents as in ET WEC Jan2012

Series Elements: Series means one after the other, like a series of events, or a series of hockey matches. A branch consists of a series of elements, and such elements are said to be in series with each other. However, there is a difference. While the players playing a series may change, the current flowing in a series path has no such possibility. The same current must flow through each of the series elements one after the other. A series path does not cross over a principal node. In other words there is no chance of current splitting or additional current joining on the way. In figure 2-9, d-a-b is a series path. Although b is a principle node, the path does not cross over it. On the other hand, path d-a-b- c is NOT a series path because of principle node b. There is no guarantee that current I3 is the same as current I1. All we know from KCL is that I1= I2+I3 Note: KVL is applicable to all closed paths- whether series or not. PP2-1 Determine if the paths b-c-d, b-c-d-a, a-b-c-d-a are series paths. Ans:y,n.n Equivalent Resistance of a Series Path

2.7 In figure 2-10, R1, R2, R3 is a series path carrying current I caused by the voltage source Vs. By KVL, summing voltage rises in a clockwise direction,: Vs-V1-V2-V3=0 Or Vs= V1+V2+V3 =IR1+IR2+IR3 =I(R1+R2+R3) Dividing both sides by I: Vs/I= R1+R2+R3 Now we should try to give some meaning to Vs/I. Vs is a voltage causing I amp in a path. Then Vs /I must be a resistance, the equivalent resistance of the path. We call this R R= R1+R2+R3 Note that in the expression for the equivalent resistance, every series resistance shows up in the sum. If there were n resistances R1, ..RxRn in series in the path, R= Example 2-4 A series path contains 10 resistors of 1, 2, 3..10, respectively. What is the equivalent resistance of the path? Solution: R= (2-7) (series) (2-7)

The resistance values in the summation form an arithmetic progression, with Rx=x R = PP2-2 What is the resistance in a series path whose element resistances form a GP, the first one being 1, and the last being 32 Ans : 63 Voltage Division Rule 2.8 In a series path what share of the total applied path voltage must exist across a certain resistor? Refer again to Figure 2-10. Vs= V1+V2+V3, and Vx=IRx, where suffix x refers to the xth resistor Vx/Vs = =(10+1) (10)/2= 55

Vx =Vs

= Vs

(2.8)

Every resistor in a series path takes a share of the applied voltage which is proportional to its resistance value. Recall for comparison that in a parallel combination, every conductor takes a share of total current which is proportional to its own conductance. Do you again recall the sharing of meat in sacrificial animal? Example 2-5 Fig 2-11 shows a voltage divider. If Vs =165 v, R0=10, R1=11, R2=12, and Rn =10+ n so that R10=20. Find the voltages V0 to V10 Solution V0=(R0/R)Vs = (10/165) 165=10 v. Similarly, V15=15 v Voltage Drop Across Open Circuit In Fig 2-11 (b) the resistance between points b and c is infinite (open circuit). What is the voltage Vbc? By Voltage Divider Rule, Vbc = 300 V =300 V! The entire applied voltage has appeared across the open circuit, even though the other resistors are also quite large! The circuit is equivalent to that in Fig 2-11 ( c ) with 300 volts across b and c. Electric Shock Now suppose a superman connects himself across b and c as shown in Fig 211(d). (May be he is charging himself!). How much voltage will appear across his body? That will depend on how much resistance is presented by his body between terminals b-c. Suppose it is 1. The voltage across his body, Vbc=300 A= 0.1 A He feels happy he is being charged. Caution! Humans are not meant to be working on electric power. Suppose a human replaced the superman across b-c. Depending on his skin quality, perspiration condition, and the contact area, he may offer a resistance of 10 k. In that case he may draw a current of =23 mA, which if at 50 Hz, will generally be sufficient to give an unforgettable shock. Doctors say it is current which kills, not voltage alone. We often see crows perching on high voltage lines. Are those actually supermen charging themselves on free electricity, or are those crows? Why dont they get electrocuted? They do! When an unfortunate crow spreads it = 0.1V, and current flowing through him will be

wings far enough to touch across two wires! As long as he is sitting on one wire only, his entire body is at a potential floating with the voltage on that wire. No current flows, and he is safe. Humans do not normally plug themselves across live terminals. Most shocks occur through unintentionally touching a live wire while standing on ground. If the voltage system has some path to ground, this may complete a circuit and result in a shock. Wear insulated boots / gloves to avoid shock! 2.9 2.9.1 Use of series Parallel combinations Standard Values

Resistors, and indeed all industrial products (even shoes), are produced in ranges of standard values agreed upon by professional bodies. Appendix A shows Standard values of resistors in various tolerances. Tolerance is the maximum allowed deviation from the nominal value. A resistor of 100 10% can have a value anywhere between 90 and 110. A circuit designer uses standard values and specifies the tolerance required for each resistor or other element. Standard values in each tolerance specification, eg, 10% have been chosen in such a way that any value specified by the designer will lie within the tolerance of at least one standard value. For example, 11 is not a standard value for 10% tolerance range. But either the standard value 10, or12 can be used with a nominal error of less than10%. Series parallel combinations of resistors can sometimes be used to obtain nonstandard values if demanded by a design. When two 100 10% resistors are connected in series, the combination value can lie anywhere between a min of (90+90) , and a maximum of (110+110) , because each resistor may deviate from the nominal value in the same direction. A parallel combination may result in a value between 90||90 , and 110||110 . 2.9.2 Circuit Solution

Simple circuits not involving a single independent source can also be solved using series-parallel combinations. Example 2-6 Find the power consumed in the 4 resistor in Fig 2-12. Solution P4=(I4)2/4 w. But I4 is not known. Since the circuit has only one source driving it, the circuit can be solved using series parallel simplifications. 2 and 3 carrying I1are in series. The two can be combined into one resistor. R2-3=2+3=5 Similarly the 2 and 6 carrying I3 are in series and can be replaced by a single resistor: R2-6=2+6=8 Also, the 5, 7, and 4 resistors are in series and hence can be combined into one resistor R5-7-4 =5+7+4=16 These three replacements are shown in Fig 2-12(a). Note that the branch currents I2, and I3 are not affected.

Now in Fig 2-12(a), the new 8, and 16 resistors are in parallel. They can be replaced by a single resistor R 16-8 =(16 x 8)/(16+8)=5.333. This is shown in Fig 2-12(b). The replacement resistor of 5.333 now carries all of I 1. All three resistors in Fig 2-12(b) are in series, with an equivalent of 11.333 . Hence, I1=9/11.333 =0.794 A.

Now we can calculate each parameter of the elements on the route of I1. For example, voltage V1 across the 1 resistor is 1 x I1=0.794 V, power=1 x 0.7942=0.630 w. Our specific task is the power in the 4 resistor (Fig 3-6) which forms part of the 16 resistor (Fig 2-12(a)) now hidden in the 5.333 equivalent resistor in Fig 212 (b). The 5.333 is the parallel equivalent of the16 and 8 resistors in Fig 2-12(a). We can find the voltage across the 5.333 resistor: Vx=5.333 I1 =4.234 V (note the polarity).

This same voltage is present across the 16 resistor of Fig 2-12(a). Hence we calculate the current I2 through the 16 resistor: I2=4.234/16=0.265 A (direction is downwards). Note: I2 could also be calculated using a current divider on I1 Now this I2 is also flowing through the desired 4 in fig 2-12. Hence: P4=4 (I2)2 Bridge Circuits 2.10 You are familiar with the Wheatstone bridge. It consists of two voltage dividers on the same voltage, and a meter bridging the dividers detects the unbalance. In a circuit model, the sensor can be represented with an equivalent resistor. The wheatstone bridge is so common that it is simply referred to as the bridge. Fig 2-13 shows R1, R3, and R2, R4 as the two voltage dividers on Vs. Rm represents the meter to read the unbalance V L-VR. VL= VR= Vs Vs. =0.280 w

The bridge is balanced, ie VL=VR when = = leading to (2.9)

This circuit is now very popular in measurement of variables, eg temperature. Any one of the four resistors forming the arms of the bridge is replaced with a resistor sensitive to the variable to be measured. Starting with a balanced

bridge, any change in the variable unbalances the bridge, and for small values of unbalance, the unbalance is proportional to the change. An uncountable number of instruments are made on this principle. The analysis of this simple circuit is not possible using series / parallel combinations. No two resistors can be said to be either in series or in parallel. But a series parallel approach can be applied after applying a transformation called Delta-Wye transformation to part of this circuit. R1, R2, and Rm in Fig 3-7 together look like a (Delta). Similarly, R3, R4, Rm also form a delta. Delta-Wye Transformations 2.11 Suppose we replace the lower delta between points L,R, and B with a circuit as shown in Figure 2-13. The replacement circuit between the same points looks like letter Y (Wye) or a star, and is known by either name. What is the condition that the voltages and currents in the overall circuit, especially at points L,R,B will not be disturbed? Simply that the effective resistances between node pairs R-B, B-R, and R-L should not change! Fig 2-14 shows the Delta (solid lines) and the proposed Y (dashed lines) superimposed between points L,R,B. Now considering the delta: RLB= RRB= RLR= R3 || (Rm+R4) R4 || (Rm+R3) Rm || (R3+R4) = = = (2.10) (2.11) (2.12)

Considering the Y: RLB=RL+RB RRB= Rr+RB RLR=RL+RR Combining (2.11), and (2.14) RL+RB = RR+RB = RL+RR = (2.16) (2.17) (2.18) (2.13) (2.14) (2.15)

Solving equations (2.16) to (2.18) simultaneously gives RL (2.19)

RR RB This gives the transformation from Delta to Wye.

(2.20) (2.21)

Notice that the formula for RL appears close to the formula for a parallel combination of R1, R2 (enclosing RL in Fig 2-15), except that the denominator includes the third resistor also in the sum.(This should help the student write these formulas without reference to the book) A special situation commonly found in electrical power calculations occurs with Rm=R3=R4=R Then RL=RR=RB=RY RY= R /3 Solving the same set of equations gives the formulae for Y- transformation. Rm= R3= R4= (2.22) (2.23) (2.24)

Again notice the symmetry. The numerator is sum of arms of the Y taken two at a time and the denominator is the arm opposite the side of the we are trying to calculate. Example 2-7 In Fig 2-13 reproduced opposite, Vs=6V, R1=1.2 K, R2=1k, R3=3 k, R4=3k, Rm=100 . Using -Y transformation, find the current flowing in Rm from left to right. Solution We can replace either . Suppose we choose the upper delta. It appears in Fig 2-16 (a) with values inserted. Fig 2-16 (b) shows the replacement Y whose resistor values have to be calculated. Then Ra=(1.2k x 1k) / (1.2k+1k+0.1k) = 0.521 k 0.043 k

Rb=(1k x0.1k) / (1.2k+1k+0.1k) =

Rc=(1.2k x0.1k) / (1.2k+1k+0.1k) =0.052 k We insert the Y in Fig 2-17 to obtain the circuit in Fig 2-17(a), and from there, 2-17(b) by combining series resistances. Again combining the parallel resistances of 3.043 k and 3.052 k in parallel we get fig 2-17 (c). Using the voltage divider rule, we get the voltage Vx across the 1.524 k as:

...Vx=6

4.470 V

This is now inserted in Fig 3-11(a) and shown as Fig 2-17(d) Using a voltage divider, VL=Vx V And VR=Vx VLR=VL-VR= ILR=-0.013/Rm =4.470 -0.013 v =-0.013V/0.1 k = -0.13mA = 4.407 V =4.470 = 4.394

2.12 Dependent sources The ideal sources we have seen till now are independent sources in that their output is fixed and not dependent on anything outside them. Look at the circuit of Figure 2-18. There is a voltage source of 10 V. This is an independent source. Similarly, the 4A source is an independent source. On the other hand, there are IDEAL sources which depend on variables outside themselves. The diamond shaped block marked 5V3 is a dependent current source because its magnitude is 5 times whatever value V3 may take. The diamond shape indicates it is a dependent source. The arrow inside indicates it is a current source. It puts out 5 Amp for every volt of V3. It is a voltage controlled current source (VCCS). We can also have a current controlled current source (CCCS) whose output current will be controlled by some specified current elsewhere. Similarly, the diamond marked 1000I0 is a voltage source (see the polarity signs inside the diamond) but has a magnitude which is 1000 volt for every 1A of the current marked I0. This is a current controlled voltage source (CCVS), and we can also have voltage controlled voltage sources (VCVS). Fig 3-2 shows all the four types. Note that the description of a controlled source is incomplete without identification of the controlling current or voltage. In Fig 32 the Ix, Vx are shown separated by a dotted line to indicate these may be parts of different circuits elsewhere. 2.13 Use of Kirchhoffs Laws in Analysis

In circuit analysis, dependent sources are treated like the corresponding independent sources. Example 2-8 Find the voltage across, current through and power consumed in each element in the circuit in Fig 2-20 Soultion We have the Ohms law, KCL, and KVL. We shall make an effort to move from knowns to unknowns by using these laws. But first we give identifiers

to each current and voltage. This has been done in Fig 2-21.Also marked are the two (inner) meshes as 1, and 2; and the two principal nodes as A, B. Now we apply KCL on node A. Imagine a circle around node A. (Fig 2-22). How many branches are crossing this circle? Three! Hence KCL equation on this node will involve three currents. We can sum either currents into the circle (or node) or currents out of it. We declare our intention by writing: (NodeA), i in Beginners are strongly advised to go round this circle and write each current as they cross it. Writing each inward current as a positive term and outward current as a negative term we write: Is I0 +10 I0=0. (2.25) This is so because we are summing currents into node A as declared above. What happens if we sum currents out of node A? First we declare the intention: (NodeA), i out Again going round the circle simply write currents outward as positive terms and those inward as ve terms. -Is+I0-10I0=0 (2.26)

Equation (2.25), and (2.26) are identical. Hence either one is sufficient. Either (1), or (2) will yield Is=-9I0 (2.27

Now we write KVL on mesh 1. We can go round the mesh either clockwise or counter clockwise, and we can sum either voltage rises or voltage falls as we go round. We must also declare a starting (and ending) point. Let this be node B. Suppose we decide to sum rises going CW. We declare: M1, CW, rises: Starting at B going CW along mesh one the 10 V source provides a rise of 10 v. Next, V2 is a fall (CW), V4 is another fall, and we have covered the mesh. Hence: 10-V2-V4=0. We can eliminate unknowns V2, V4, by using Ohms law. V2= 2 Is volts, and V4=4 I0 volts. Inserting, we get: 10-2Is-4 I0=0 (2.28)

Again we can use equation (2.27) to write: 10 -2(-9I0) -4I0=0, yielding:

I0=10/14 amp.=5/7 =0.714A. Then: Is=-9 I0=-45/7 A=-6.429 A V2= 2 Is= -6.429 x 2= -12.858 v What does the ve sign mean? Simply that we decided to measure the voltage left end of 2 with respect to its right end. Actually, the left end is at 12.585 volts BELOW the right end! Nothing wrong! V3=(-10 I0) x 3 =-21.428 v. V4=4 I0 =2.855 v To find Vo, apply KVL on mesh 2. M2, CW, rises: V4-V3-Vo=0 2.885-(-21.428)-Vo =0 Vo=18.543 v Power consumed in the 10 volt source =P(10v)= -10 x Is (Is is flowing out of +ve terminal)

=-10 x (-6.429)= +64.29 W. Although Is is flowing out of +ve terminal, but since Is has a ve value, power is consumed in the 10 volt source. P2= (Is)2 R=(-6.429)2 x 2 =82.664 W

P2 can be calculated as vi product, but beware of the signs. Power in a resistor will always be positive (consumed). PP 2-3 (a) Find Io, Vo, V2, V3 and the power delivered by the 10 v source. (b) Replace the 10v source with a zero volt source (short circuit). Find Vo, V2, V3. (c) What happened? Ans: (a) Io=0.667A (b) All zero (c) No independent source of energy in the circuit. A dependent source is not an independent source of energy.

Inter Node Currents

2.15 Look at Fig 2-23. It has principal nodes a, b, c, and d, which have voltages with respect to earth of Va through Vd respectively. Current Iac flows from node a to node c. Similarly, other nodal currents are:I ab, Iad, Ibc, Ibd, Icd. Knowing Va and Vb it is possible to find Iab, if the values of the sources and the resistors are known. Iab= Iab= A = (Ohms Law)

Iac is slightly more involved. Vab is the drop from a to b Vab=-2+1 Iac+2 Iac+3 Iac+4 Iac Va-Vb+2=Iac (1+2+3+4)= 10 Iac Iac= A =

Similarly, Vad= Va-Vd=1+7 Iac Iac= A =

It appears in each case above that Ixy= Amp, where x, y are any two nodes in the circuit.

Using this rule we try to write Ibc directly Ibc= Ibd= Icd= A A A

As Va through Vd are known all currents are known. How to find Va through Vd is an issue we will tackle in the next chapter.

Problems Sections 2.1 to 2.3 P-2-1 In fig P-2-1,I1= -3A, I2=2A, I3=4A, Ix=-6A. Find I Ans: -3A P-2-2 In fig P-2-2 find Ix if I1= +3A, I2=-2A, I3=-4A, I=3A Ans: 6A P-2-3 In Fig 2.2 find Ix P-2-4 In Fig P-2-7, Find Ix, Iy in terms of I1,I2. Section 2.4 P-2-5 Three resistors of 2 k, 3k, and 6k are connected in parallel. What is the equivalent resistance of the parallel combination? P-2-6 A resistance of 1.1K is required whereas resistors of 1 k, 1.2 , 1.8k, and 100 k are available. Design a parallel combination using two of these resistors which is closest to the required value. Hint : 1 k resistor will not be part of the combination. Section 2.5 P-2-7 In figure P-2-3, use current division rule to find Ix, Iy. P-2-8 In fig P-2-4, Ix=2A. Use current division rule to find the: a) Iy b) Right to left voltage across the 2 resistor. Ans:2/3 amp, -5.33 v

P-2-9 In fig P-2-4, Iy=2A. Find the: a) Ix b) Right to left voltage across the 2 resistor. Ans:2/3 amp, -5.33 v P-2-10 Find Ix in fig P-2-5 Section 2.6 to 2.8 P-2-11 Apply KVL and Ohms Law to Fig P-2-6 and find Vx P-2-12 In fig P-2-5 Use KVL find the voltage across the 4 resistor. Ans:8v, top end +ve P-2-13 In fig P-2-7 a) b) c) d) P-2-14 In the KL Practice circuit in fig P-2-8, apply V1=1v across R1 with top end +ve. Find and mark all voltages and currents on a copy of the diagram. Write KVLover the left mesh of the circuit in terms of Vs, I1, Ix Write KVL over the right mesh of the circuit in terms of Vs, I 2, Iy In the above two equations insert Ix=I1-I2, and Iy=-Ix=I2-I1 Solve for I1, I2

P-2-15 In the KL Practice circuit in fig P-2-8, apply V1=1v across R1 with bottom end +ve. Find and mark all voltages and currents on a copy of the diagram. Compare the answers with those in P-2-15

Sec 2.9,

P-2-16 A resistor of 1000, 10% is connected in parallel with a resistor of 100 1%. What are the possible min and max values of the combination? Ans: 89.2, 92.5 P-2-17 In Fig P-2-17 find a) The resistance seen by the source b) Vx c) The power consumed in the 20 resistor P-2-18 In Fig P-2-18 find a) The resistance seen by the source b) Vx c) The power consumed in the 20 resistor

Secs 2.10, 2.11 P-2-19 a) Convert the delta of fig P-2-19 (a) to an equivalent star. an

b) Convert the star of fig P-2-19 (b) to equivalent delta. P-2-20 Find Vx in fig P-2-20 P-2-21 Find Vx in fig P-2-21 P-2-22 Find Vx in fig P-2-22 P-2-23 Find Vx in fig P-2-23 P-2-24

Find Vx in fig P-2-24

Sec 2.12, 2.13 P-2-25 (a) Find Is, Io in Fig P-2-25. (b) The resistance seen by a source is defined as its voltage divided by the current supplied by it. Find the resistance seen by the source in Fig P-2-25

P-2-26 The resistance seen by a source is defined as its voltage divided by the current supplied by it. Find the resistance seen by the source in Fig P-2-26

P-2-27 The resistance seen by a source is defined as its voltage divided by the current supplied by it. Find the resistance seen by the source in Fig P-2-27

P-2-28 The resistance seen by a source is defined as its voltage divided by the current supplied by it. Find the resistance seen by the source in Fig P-2-28 Section 2-14 P2-29 Assuming node voltages are known. Write expressions for all inter currents in Fig P-2-29 nodal

P2-30 Assuming node voltages are known. Write expressions for

all inter nodal currents in Fig P-2-30 P2-31 Assuming node voltages are known. Write expressions for all inter nodal currents in Fig P-2-31 Additional Problems P-2-32 A galvanometer with full scale deflection current of 50A and a resistance of 500 is to be used to measure upto 1 A by using it in a current divider. What resistance must be placed in shunt with it so that the meter draws 50A when the parallel combination is fed with 1A total. What will be the drop across the galvanometer when it is reading full scale? P-2-32 A galvanometer reads full scale when 50 mv is applied across it. It has a resistance of 500. This galvanometer is to be used to measure 1000 volts by using it in a voltage divider. What resistance must be placed in series with it so that it reads full scale when 1000 volts are applied to the divider? P-2-33 A voltmeter capable of reading 1000 v and having a resistance of 100k is to be used to measure the output of a voltage divider as shown in Fig P-2-33. (a) (b) (c) (d) Calculate the output of the divider without the meter connected. What voltage will the meter actually read if it is accurate? What will be the reading if a voltmeter of 1 M is used instead? Will the reading be less or more than the value in part (a)?

Note: This reduction in voltage is due to loading. The meter resistance acts as a load on the divider, draws current, and reduces voltage at the points of measurement. The effect will be less with a voltmeter having higher resistance.

P-2-34 In Fig P-2-34(a) and (b) find Io using a current divider .

You might also like